Difference between revisions of "2002 AIME I Problems/Problem 4"

m
(Problem)
Line 1: Line 1:
 
{{empty}}
 
{{empty}}
 
== Problem ==
 
== Problem ==
 +
Consider the sequence defined by <math>a_k =\dfrac{1}{k^2+k}</math> for <math>k\geq 1</math>. Given that <math>a_m+a_{m+1}+\cdots+a_{n-1}=\dfrac{1}{29}</math>, for positive integers <math>m</math> and <math>n</math> with <math>m<n</math>, find <math>m+n</math>.
  
 
== Solution ==
 
== Solution ==

Revision as of 16:27, 25 September 2007

This is an empty template page which needs to be filled. You can help us out by finding the needed content and editing it in. Thanks.

Problem

Consider the sequence defined by $a_k =\dfrac{1}{k^2+k}$ for $k\geq 1$. Given that $a_m+a_{m+1}+\cdots+a_{n-1}=\dfrac{1}{29}$, for positive integers $m$ and $n$ with $m<n$, find $m+n$.

Solution

This problem needs a solution. If you have a solution for it, please help us out by adding it.

See also